Surgery (50q).1

50 Questions | Attempts: 504
Share

SettingsSettingsSettings
Surgery Quizzes & Trivia

Questions and Answers
  • 1. 

    Ten days after undergoing liver transplantation, a patient's levels of gamma-glutamyl transferase (GGT), alkaline phosphatase, and bilirubin begin to rise. Which of the following is the most appropriate next step in diagnosis? 

    • A.

      Measurement of preformed antibody levels

    • B.

      Ultrasound of biliary tract and Doppler studies of the anastomosed vessels

    • C.

      Liver biopsy and determination of portal pressures

    • D.

      Liver biopsy and more detailed liver function tests

    • E.

      Liver biopsy and trial of steroid boluses

    Correct Answer
    B. Ultrasound of biliary tract and Doppler studies of the anastomosed vessels
    Explanation
    In all other solid organ transplants, deterioration of function 10 days out would suggest an acute rejection episode, and appropriate biopsies would be done to confirm the diagnosis. In the case of the liver, however, antigenic reactions are less common, whereas technical problems with the biliary and vascular anastomosis are the most common cause of early functional deterioration. They are, therefore, the first anomalies to be sought. Preformed antibodies (choice A) are responsible for hyperacute rejection, which would be evident within minutes of establishing blood flow to the graft. Choices C, D, and E are centered on liver biopsy, which would be done only after technical problems have been ruled out.

    Rate this question:

  • 2. 

    A previously healthy, intoxicated, 19-year-old man is driving a car without using a seat belt. He crashes the car into the back of a parked truck. In the process he slams his abdomen into the steering wheel and ruptures his spleen. Which of the following is the most important problem associated with this type of injury?

    • A.

      Bacteremia

    • B.

      Electrolyte abnormalities

    • C.

      External blood loss

    • D.

      Internal blood loss

    • E.

      Peritonitis

    Correct Answer
    D. Internal blood loss
    Explanation
    The spleen is a highly vascularized organ, and is vulnerable to traumatic rupture. This can occur "spontaneously" (i.e. with minimal trauma such as falling against a table or even overly vigorous palpation during a physical examination) in patients with an enlarged spleen due to disease (e.g., leukemias, autoimmune diseases with red cell sequestration in the spleen, or as a complication of portal hypertension). Alternatively, splenic rupture can occur in previously normal individuals who have severe trauma to the abdomen. In either case, the heavily vascularized spleen is usually unable to stop (often massively) bleeding internally. Emergency splenectomy is indicated to control the bleeding. Bacteremia (choice A) and peritonitis (choice E) are much less of a risk in splenic rupture than in rupture of a hollow viscus such as the colon, since the spleen is usually sterile. Electrolyte abnormalities (choice B) can develop secondarily to the ischemia produced by severe blood loss; these are much less critical than the blood loss itself and will often correct spontaneously with adequate replacement of blood. External blood loss (choice C) is often insignificant in injuries such as this.

    Rate this question:

  • 3. 

    After suitable calculations have been made using the modified Parkland formula, a 70-kg man with extensive third-degree burns is receiving Ringer's lactate at the calculated rate, which happens to be 750 mL/hr. The infusion was started within 30 minutes of the time when the burn occurred. Over the next 3 hours, his urinary output is recorded as 15 mL, 22 mL, and 18 mL. It is verified that the Foley catheter is open and draining freely. The urine is dark yellow, without blood, and has a specific gravity of 1040 and a sodium concentration of 10 mEq/L. The patient's blood pressure is 100/70 mm Hg, his pulse is 98/min, and his central venous pressure is 2 cm H2O. On the basis of these findings, which of the following is the most appropriate next step in management?

    • A.

      Diuretics should be given

    • B.

      Fluid administration should continue at the present rate

    • C.

      The rate of fluid administration should be decreased

    • D.

      The rate of fluid administration should be increased

    • E.

      Treatment is needed for renal failure

    Correct Answer
    D. The rate of fluid administration should be increased
    Explanation
    The calculations made by standard formulas are only an educated guess. Once fluid administration begins, we judge its adequacy by the information provided by urinary output and central venous pressure, aiming for an output of 1-2 mL/kg/hr, while not exceeding a venous pressure of 10 or 15. In this case, our calculations fell short of the mark, and the patient needs more fluids at a faster rate. Diuretics (choice A) are not the answer when all indicators show fluid need: his venous pressure is low, his blood pressure and pulse rate are marginal, and he has very concentrated urine. He needs fluids! The present rate (choice B) may follow the "formula," but it is clearly inadequate. Decreasing the rate (choice C) is the very opposite of what is needed. And as for renal failure (choice E), it is indeed part of the differential diagnosis whenever urinary output is not as high as it should be. However, the vignette told you that his urine is highly concentrated and has well less than 20 mEq/L of sodium: evidence of superb kidneys trying to conserve fluid to the best of their ability. Don't blame them.

    Rate this question:

  • 4. 

    A 27-year-old immigrant from El Salvador has a 14 × 12 × 9 cm mass in her left breast. It has been present for 7 years and has slowly grown to its present size. The mass is firm, nontender, rubbery, and completely movable, and it is not attached to the overlying skin or the chest wall. There are no palpable axillary nodes or skin ulceration. Which of the following is the most likely diagnosis? 

    • A.

      Breast cancer

    • B.

      Chronic cystic mastitis

    • C.

      Cystosarcoma phyllodes

    • D.

      Intraductal papilloma

    • E.

      Mammary dysplasia

    Correct Answer
    C. Cystosarcoma phyllodes
    Explanation
    Cystosarcoma phyllodes occurs in young women, grows to huge size over many years, and yet spares the skin, the nodes, and the underlying chest wall. There is no particular connection with Central America, but often these are seen in immigrants of limited financial circumstances, who have had no access to medical care in their own countries. Breast cancer (choice A) this big and for these many years, would have ulcerated the skin, would be fixed to the chest wall, and would have produced massive axillary metastasis. Chronic cystic mastitis (choice B), also known as mammary dysplasia (choice E), is seen in women of reproductive age, who complain of tender and lumpy breasts related to the menstrual cycle. Large cysts can develop in this disease, but not to the huge size described in the vignette. Intraductal papilloma (choice D) is the most common source of bleeding from the nipple. These tumors are tiny, just a few millimeters in diameter.

    Rate this question:

  • 5. 

    A 32-year-old woman in the 2nd month of pregnancy is found to have a 5-cm mass in the upper outer quadrant of her left breast. Mammogram shows no other lesions, and core biopsy reveals infiltrating ductal carcinoma. Which of the following would be the best course of action at this time? 

    • A.

      Chemotherapy now, deferring surgery until after delivery

    • B.

      Radiation therapy now, deferring surgery until after delivery

    • C.

      Lumpectomy and axillary sampling, followed in 6 weeks by radiotherapy

    • D.

      Modified radical mastectomy now, deferring systemic therapy until later

    • E.

      Immediate therapeutic abortion and palliative breast surgery

    Correct Answer
    D. Modified radical mastectomy now, deferring systemic therapy until later
    Explanation
    The treatment of breast cancer in a pregnant woman should be the same as that in a nonpregnant woman, except for two restrictions: no chemotherapy during the first trimester, and no radiation therapy during the pregnancy. It is not necessary to terminate the pregnancy. The preferred treatment for a 5-cm tumor would be mastectomy (too big for lumpectomy). Should axillary nodes be positive, systemic therapy should be done later. Although the appropriate surgery can be done during the pregnancy, neither chemotherapy (choice A) nor radiation therapy (choice B) would be acceptable at this time. Lumpectomy (choice C) is not a good idea for a 5-cm tumor. Furthermore, the radiation therapy that must follow lumpectomy could not be given in 6 weeks, while she is still pregnant. Finally, let's not terminate a pregnancy that is doing no harm. Therapeutic abortion (choice E) does not help with the treatment of breast cancer. Neither is the pregnant woman who gets cancer of the breast automatically incurable, and thus only fit for a palliative procedure.

    Rate this question:

  • 6. 

    In the course of a robbery, a young woman is stabbed repeatedly. On arrival at the emergency department, she is shivering and asks for a blanket and a drink of water; she is noted to be pale and perspiring. Her blood pressure is 72/50 mm Hg and her pulse is 130/min. Her neck and forehead veins are large and distended. A quick initial survey reveals entry wounds in her left chest and upper abdomen. She has bilateral breath sounds and a scaphoid, nontender abdomen. As IV infusions of Ringer's lactate are started, her systolic blood pressure drops further to 40 mm Hg, no distal pulses can be felt, and she loses consciousness. Her central venous pressure at that time is 28 cm H2O. Which of the following is the most appropriate next step in management?

    • A.

      Chest x-ray to direct further therapy

    • B.

      Bilateral chest tubes

    • C.

      Diagnostic peritoneal lavage

    • D.

      Evacuation of the pericardial sac

    • E.

      Crash laparotomy in the emergency department to clamp the aorta

    Correct Answer
    D. Evacuation of the pericardial sac
    Explanation
    The diagnosis of pericardial tamponade should be obvious. The patient has the type of chest wound that can produce it, and the very high central venous pressure to prove it. Evacuation of the blood that is preventing normal ventricular filling will produce instant improvement. Later, she will need repair of the heart wound that is probably the source of the pericardial blood and may also need exploratory laparotomy. A chest x-ray (choice A) would never be ordered in a dying patient. This patient is in trouble, and she needs instant action based on a clinical diagnosis. She would die while waiting for an x-ray. Chest tubes (choice B) have nothing to offer when there are bilateral breath sounds. In this case, the patient probably does not have a tension pneumothorax to account for the shock and the high venous pressure. Diagnostic peritoneal lavage (choice C) assumes that the reason for the deterioration is intra-abdominal bleeding. With a very high central venous pressure, it is not a reasonable conclusion. Clamping the aorta (choice E) assumes that she is bleeding to death. She may be bleeding, but if that were the cause of her present predicament, her central venous pressure would be zero, or near zero.

    Rate this question:

  • 7. 

    A 62-year-old man reports an episode of gross, painless hematuria. There is no history of trauma, and further questioning determines that he had total hematuria, rather than initial or terminal hematuria. The man does not smoke and has had no other symptoms referable to the urinary tract. He has no known allergies. Physical examination, including rectal examination, is unremarkable. His serum creatinine is 0.8 mg/dL, and, except for the presence of many red cells, his urinalysis is normal and shows no red cell casts. His hematocrit is 46%. Which of the following is the most appropriate initial step in the workup? 

    • A.

      Coagulation studies and urinary cultures

    • B.

      Intravenous pyelogram (IVP) and cystoscopy

    • C.

      PSA determination and prostatic biopsies

    • D.

      Sonogram and CT scan of both kidneys

    • E.

      Retrograde cystogram and pyelograms

    Correct Answer
    B. Intravenous pyelogram (IVP) and cystoscopy
    Explanation
    Although most patients with hematuria have benign disease, silent hematuria can be due to renal, ureteral, or bladder cancer, and these malignant processes must be effectively ruled out. Intravenous pyelogram (IVP) will visualize kidney and ureteral tumors, but is not reliable enough to rule out bladder cancer. Direct visualization of the bladder mucosa by cystoscopy is the only way to rule out bladder cancer. Thus, both procedures are needed. Assuming hematuria to be a manifestation of clotting problems or infection (choice A) is unwarranted as the first diagnostic consideration in the absence of a history suggestive of such problems. Prostatic cancer can produce hematuria when it is advanced, but typically it does not show up that way in a previously asymptomatic patient. At age 62, this man needs a PSA, but this test, along with prostatic biopsies (choice C), would do nothing to find the source of the hematuria. In patients with allergy to the IVP dye, or with a creatinine above 2 mg/dL (neither of which are present here), the IVP cannot be done. In those cases, sonogram or CT scan (choice D) would provide an alternative way to look at the kidneys. The bladder would still remain as a potential site of undiagnosed cancer. Retrograde studies (choice E) are invasive and unwarranted here. A bladder full of dye will not necessarily reveal the presence of a shallow bladder cancer. The collecting system outlined by radiopaque material would not show the renal parenchyma.

    Rate this question:

  • 8. 

    A 63-year-old man, who weighs 65 kg, is in his 2nd postoperative day after an abdominoperineal resection for cancer of the rectum. An indwelling Foley catheter was left in place after surgery. The nurses are concerned because, even though his vital signs have been stable, his urinary output in the past 2 hours has been zero. In the preceding 3 hours, they had collected 56 mL, 73 mL, and 61 mL. Which of the following is the most likely diagnosis? 

    • A.

      Acute renal failure

    • B.

      Damage to the bladder during the operation

    • C.

      Damage to the ureters during the operation

    • D.

      Dehydration

    • E.

      Plugged or kinked catheter

    Correct Answer
    E. Plugged or kinked catheter
    Explanation
    In the presence of normal perfusion pressure, biological problems do not suddenly drive the urinary output from normal to zero. Such a change is invariably due to a mechanical problem. Acute renal failure (choice A) does not result in a urinary output of zero. Some urine is still produced, although it is a small volume, on the order of 5 or 10 mL per hour. Intraoperative damage to the bladder (choice B) or the ureters (choice C) would have become obvious immediately after the operation. Dehydration (choice D) would have produced a gradual decline in the urinary volume. The 3 hours preceding the onset of the problem had shown normal values (about 1 mL per kg of body weight per hour), with no downward trend

    Rate this question:

  • 9. 

    A 37-year-old woman undergoes a lumpectomy and axillary dissection for a 3-cm infiltrating ductal carcinoma, diagnosed by core biopsies, located on the upper outer quadrant of her left breast. The pathology report of the surgical specimen is received 3 days after the operation. It indicates that all margins around the tumor are clear, and that 4 of 17 axillary lymph nodes have metastatic tumor. The tumor is reported to be estrogen and progesterone receptor negative. Which of the following should further therapy most likely include?

    • A.

      Antiestrogen medication (tamoxifen)

    • B.

      Conversion to modified radical mastectomy

    • C.

      Radiation to the remaining left breast

    • D.

      Radiation to the remaining left breast and systemic chemotherapy

    • E.

      Radiation to both breasts and tamoxifen

    Correct Answer
    D. Radiation to the remaining left breast and systemic chemotherapy
    Explanation
    Lumpectomy alone has an unacceptably high incidence of local recurrence, which can be significantly reduced by radiation therapy. In addition, the presence of metastatic disease in the axillary nodes requires systemic therapy. As a rule, chemotherapy is preferred for premenopausal women, which this woman is, but it is also indicated here because she is not receptor positive. Antiestrogens alone (choice A) would not reduce the likelihood of local recurrence, and it would not help much with systemic disease because she is premenopausal and receptor negative. Conversion to mastectomy (choice B) is not needed because her surgical margins are clear of tumor. Radiation alone (choice C) would not suffice because her positive axillary nodes require the addition of systemic therapy. Radiation to the opposite breast (choice E) is not required in any event, and tamoxifen is the wrong drug for a premenopausal woman who had a receptor negative tumor.

    Rate this question:

  • 10. 

    A 45-year-old man with alcoholic cirrhosis is bleeding from a duodenal ulcer. He has required 6 units of blood over the past 8 hours, and all conservative measures to stop the bleeding, including irrigation with cold saline, IV vasopressin, and endoscopic use of the laser have failed. He is being considered for surgical intervention. Laboratory studies done at the time of admission, when he had received only one unit of blood, showed a bilirubin of 4.5 mg/dL, a prothrombin time of 22 seconds, and a serum albumin of 1.8 g/dL. He was mentally clear when he came in, but has since then developed encephalopathy and is now in a coma. Which of the following best describes his operative risk?

    • A.

      Acceptable as he now is

    • B.

      Amenable to improvement if he receives vitamin K

    • C.

      Amenable to improvement if he is given albumin

    • D.

      Prohibitive unless he is dialyzed to normalize his bilirubin

    • E.

      Prohibitive regardless of attempts to improve his condition

    Correct Answer
    E. Prohibitive regardless of attempts to improve his condition
    Explanation
    The studies show extremely marginal liver function, which would be tipped into overt liver failure by an anesthetic and an operation. He is not a surgical candidate. Choice A obviously misses the gravity of his situation. Vitamin K (choice B) works only when there is a functioning liver that can use it. In the absence of adequate liver function, it will not correct the prothrombin time. Albumin (choice C) can be given, but it will have a short life span and will not correct the liver dysfunction. The low albumin is not the main problem per se, it is a symptom of how bad his liver is. The same is true of bilirubin (choice D). It is a symptom, not the problem. We can operate on patients with much higher bilirubin if it is not due to intrinsic liver disease.

    Rate this question:

  • 11. 

    A 55-year-old woman falls in the shower and hurts her right shoulder. She shows up in the emergency department with her arm held close to her body, but the forearm rotated outward as if she were going to shake hands. She is in pain and will not move the arm from that position. Her shoulder looks "square" in comparison with the rounded unhurt opposite side, and there is numbness in a small area of her shoulder over the deltoid muscle. Which of the following is the most likely diagnosis?

    • A.

      Acromioclavicular separation

    • B.

      Anterior dislocation of the shoulder

    • C.

      Fracture of the upper end of the humeral shaft

    • D.

      Posterior dislocation of the shoulder

    • E.

      Scapular fracture

    Correct Answer
    B. Anterior dislocation of the shoulder
    Explanation
    Anterior dislocation of the shoulder is the most common dislocation of that joint. The position is classic, as is the lack of the rounded contour of the humeral head. The area of numbness represents injury to the axillary nerve, a common complication of anterior dislocation of the shoulder. Acromioclavicular separation (choice A) would be characterized by very localized pain at that particular spot and none of the other features described here. Fracture of the humeral shaft (choice C) would likewise lack the specific deformity, inasmuch as the humeral head would still be in place to provide the normal rounded contour. Posterior dislocation (choice D) typically occurs following massive uncoordinated muscle contractions (electrical injuries, epileptic seizures), and the arm and forearm are held in a more "normal" protective position, close to the body. Scapular fracture (choice E) happens only with extremely severe chest trauma; it would not happen by falling in the shower. Along with two other injuries (fracture of the sternum or fracture of the first rib), scapular fracture, when present, indicates that very severe trauma has occurred, and it is a useful clinical clue to look for hidden internal injuries.

    Rate this question:

  • 12. 

    A 22-year-old convenience store clerk is shot once with a .38 caliber revolver. The entry wound is in the left midclavicular line, 2 inches below the nipple. There is no exit wound. He is hemodynamically stable. A chest x-ray film shows a small pneumothorax on the left, and demonstrates the bullet to be lodged in the left paraspinal muscles. In addition to the appropriate treatment for the pneumothorax, which of the following will this patient most likely need?

    • A.

      Barium swallow

    • B.

      Bronchoscopy

    • C.

      Extraction of the bullet via local back exploration

    • D.

      Extraction of the bullet via left thoracotomy

    • E.

      Exploratory laparotomy

    Correct Answer
    E. Exploratory laparotomy
    Explanation
    Although this vignette describes a gunshot wound of the chest, we must remember that the chest and the abdomen are not stacked up like pancakes. There is a dome - the diaphram - that separates them, and thus an area where chest and abdomen overlap. Any gunshot wound below the nipples involves the abdomen, and such is the case here. The management of all gunshot wounds of the abdomen requires exploratory laparotomy. Barium swallow and bronchoscopy (choices A and B) are indicated if there are signs suggestive of injury to those organs (coughing up blood, spitting up blood), or if the anatomic trajectory of the bullet puts the track in their vicinity. Here, we have an entry wound on the left and a bullet lodged on the left: the midline has not been crossed. Taking out the bullet (choices C and D) is unnecessary if the missile is not pressing on some vital structure. A bullet embedded in a muscle can be left there.

    Rate this question:

  • 13. 

    A 68-year-old man is brought to the emergency department with excruciating back pain that began suddenly 45 minutes ago. The pain is constant and is not exacerbated by sneezing or coughing. He is diaphoretic and has a systolic blood pressure of 90 mm Hg. There is an 8-cm pulsatile mass deep in his epigastrium, above the umbilicus. A chest x-ray film is unremarkable. Two years ago, he was diagnosed with prostatic cancer and was treated with orchiectomy and radiation. At that time, his blood pressure was normal, and he had a 6-cm, asymptomatic abdominal aortic aneurysm for which he declined treatment. Which of the following is the most likely diagnosis?

    • A.

      Dissecting thoracic aortic aneurysm

    • B.

      Fracture of lumbar pedicles with cord compression

    • C.

      Herniated disc

    • D.

      Metastatic tumor to the lumbar spine

    • E.

      Rupturing abdominal aortic aneurysm

    Correct Answer
    E. Rupturing abdominal aortic aneurysm
    Explanation
    Abdominal aortic aneurysms have a high incidence of rupture once they reach or exceed a size of 6 cm. Often, the first manifestation is excruciating back pain, as the blood leaks into the retroperitoneal space before the aneurysm blows out into the peritoneal cavity. The combination of a big aneurysm and sudden severe back pain should always lead to this presumptive diagnosis. Looking for orthopedic or neurologic explanations can be a deadly mistake. Dissecting thoracic aortic aneurysm (choice A) could also cause excruciating back pain, but the pain usually starts as retrosternal and later migrates down. The absence of hypertension mitigates against this diagnosis, and one would expect to see a wide mediastinum on the chest x-ray film. Fracture of the spine with cord compression (choice B) could indeed happen to someone who recently had prostatic cancer, but the symptoms would be primarily neurologic deficits from cord compression. The pain from a herniated disc (choice C) runs down the leg and is exacerbated by sneezing and coughing. Metastatic tumor (choice D) is a good bet in someone with prostatic cancer. However, the pain of bony metastasis is present for weeks or months, and is constant, dull, low grade, and worse at night-not the sudden excruciating pain of this vignette.

    Rate this question:

  • 14. 

    A middle-aged homeless man is brought to the emergency department because of very severe pain in his forearm. He had passed out after drinking a bottle of cheap wine, and then slept on a park bench for an indeterminate time, probably more than 12 hours. Shortly after he woke up and began to walk, the pain began. There are no signs of trauma, but the muscles in his forearm are very firm and tender to palpation, and passive motion of his fingers and wrist elicits excruciating pain. Pulses at the wrist are normal. Which of the following is the most appropriate next step in management

    • A.

      Analgesics and observation

    • B.

      Immobilization in a sling

    • C.

      Immobilization in a plaster cast

    • D.

      Emergency embolectomy

    • E.

      Emergency fasciotomy

    Correct Answer
    E. Emergency fasciotomy
    Explanation
    The presentation is classic for compartment syndrome, triggered by prolonged ischemia followed by reperfusion (the arm pressed against the park bench until he woke up and changed position), and located in one of the two most common sites (forearm and lower leg). He has the most reliable physical finding (pain on passive extension), and the diagnosis is not ruled out by normal pulses. Only a fasciotomy will solve his problem. Analgesics and observation (choice A) will result in permanent damage to the compartment muscles. Immobilization, by sling (choice B) or cast (choice C), will allow the high pressure within the compartment to continue to destroy the muscles. Embolectomy (choice D) assumes an arterial occlusion, which his normal pulses rule out.

    Rate this question:

  • 15. 

    A 54-year-old obese man gives a history of burning retrosternal pain and heartburn that is brought about by bending over, wearing a tight belt, or lying flat in bed at night. He gets symptomatic relief from over-the-counter antiacids or H2 blockers, but has never been formally studied or treated. The problem has been present for many years and seems to be progressing. Which of the following is the most appropriate next step in management?

    • A.

      Barium swallow

    • B.

      Cardiac enzymes and ECG

    • C.

      Proton pump inhibitors

    • D.

      Endoscopy and biopsies

    • E.

      Laparoscopic Nissen fundoplication

    Correct Answer
    D. Endoscopy and biopsies
    Explanation
    The clinical picture is fairly convincing for long-standing gastroesophageal reflux. The main concern is the degree of peptic esophagitis that he may have developed, and the possibility of Barrett's esophagus and premalignant changes. Endoscopy and biopsies will provide the answer. Barium swallow (choice A) would provide anatomic evidence of hiatal hernia and evidence of reflux, but would not tell us whether Barrett's esophagus has developed. Cardiac enzymes and ECG (choice B) would be part of the work-up (along with pH monitoring) if we were uncertain as to the genesis of ill-defined low retrosternal and upper epigastric pain. This man gives a classic presentation for reflux. Proton pump inhibitors (choice C) might likewise be indicated for this man, but not until we know the severity and potential premalignant stage of his disease. Nissen fundoplication (choice E) may some day be needed here, but one would not jump to a surgical solution based only on a clinical presentation.

    Rate this question:

  • 16. 

    A pedestrian is hit by a car and knocked unconscious. Within a few minutes, he starts to move around and moan. When the ambulance arrives, he is moving all four extremities and mumbling that his neck hurts. Shortly thereafter, he lapses again into a deep coma. In the emergency department, it is noted that his left pupil is fixed and dilated, and he has clear fluid dripping from the left ear. The trauma team intubates him nasally over a fiberoptic bronchoscope and does a quick initial survey that reveals no other obvious injuries. He is hemodynamically stable. Which of the following is the most appropriate next step in management? 

    • A.

      Antibiotics and high dose corticosteroids

    • B.

      Cervical spine and skull x-ray films

    • C.

      CT scan of the head, extended to include the cervical spine

    • D.

      Otoscopic examination and laboratory studies of the fluid

    • E.

      Emergency ear surgery to stop the leak of cerebrospinal fluid

    Correct Answer
    C. CT scan of the head, extended to include the cervical spine
    Explanation
    Clinically, this man has a life-threatening head injury, with a high probability that he may have an intracranial hematoma that has to be drained. CT scan is the only study that will show such a hematoma. Furthermore, we know that the head trauma was severe enough to produce a fracture of the base of the skull (cerebrospinal fluid dripping from the ear); thus, it may well have produced injury of the cervical spine as well. This is likely since he was complaining of neck pain, and it is imperative that it be diagnosed to protect his cord, which is probably still intact (when he was last awake he still was moving all four extremities). The most expedient way to do it is to extend the CT scan to include the neck. Antibiotics and steroids (choice A) are not indicated. The former used to be given for cerebrospinal fluid leaks, but is no longer considered appropriate. Steroids are used if the cord is injured, but we have reason to believe that it is still intact. Cervical spine x-ray films (choice B) are a good idea, but skull x-ray films are not. If his only problem were the cervical spine, we would indeed go for the x-rays. But we also have to check his head, for which we need the CT. Let the CT take care of both issues. The same is true of choice
    D. We do not need to look into the ear or to study the fluid. The CT will show the fracture to the base of the skull, at the same time that it will tell us if an intracranial hematoma has to be drained. Cerebrospinal fluid leaks caused by fractures to the base of the skull typically stop spontaneously. Surgery is rarely needed for them. When it is needed, it is not an emergency and would not be done through the ear. Thus, choice E is wrong on all counts.

    Rate this question:

  • 17. 

    While working at a bookbinding shop, a young man suffers a traumatic amputation of his index finger. The finger was cleanly severed at its base. The patient and the finger are brought to a first-aid station, from which both are to be transported to a highly specialized medical center for replantation to be done. Which of the following is the correct way to prepare and transport the severed finger?

    • A.

      Dry the finger of any traces of blood and place it in a cooler filled with crushed ice

    • B.

      Freeze it as quickly as possible, and transport it immersed in liquid nitrogen

    • C.

      Immerse it in cold alcohol for the entire trip

    • D.

      Paint it with antiseptic solution and place it on a bed of dry ice

    • E.

      Wrap it in a moist gauze, place it on a plastic bag, and place the bag on a bed of ice

    Correct Answer
    E. Wrap it in a moist gauze, place it on a plastic bag, and place the bag on a bed of ice
    Explanation
    The digit must be kept from drying out, must not be injured with any chemical agents, and must not be placed in direct contact with ice or allowed to freeze. Direct contact with ice (choice A) is one of the damaging events to be avoided. Freezing (choice B) is absolutely contraindicated. Alcohol (choice C) would damage the tissues. Antiseptic solutions and direct contact with dry ice (choice D) would damage the finger both chemically and physically.

    Rate this question:

  • 18. 

    An out-of-shape, recently divorced, 42-year-old man is trying to impress a young woman by challenging her to a game of tennis. In the middle of the game, a loud "pop" (like a gunshot) is heard, and the man falls to the ground clutching his ankle. He limps off the court with pain and swelling in the back of the lower leg. Although he can still weakly plantar-flex his foot, he seeks medical help the next day because of persistent pain, swelling, and limping. He can put weight on that foot with no exacerbation of the pain, but the motion of taking a step is painful. Which of the following would be the most likely finding on physical examination?

    • A.

      Tapping on the calcaneus is extremely painful

    • B.

      The ankle joint can be abducted farther out than the normal contralateral side

    • C.

      The ankle joint can be adducted farther in than the normal contralateral side

    • D.

      There is a gap in the Achilles tendon easily felt by palpation

    • E.

      There is crepitation and grating by direct palpation over either malleoli

    Correct Answer
    D. There is a gap in the Achilles tendon easily felt by palpation
    Explanation
    The clinical description is that of a rupture of the Achilles tendon. The injured structure is so close to the skin that direct palpation of the gap in the tendon is usually possible. A fracture of the calcaneus (implied in choice A) would happen with a fall from a height, landing on one's feet. The ability to bend a joint beyond the normal boundaries (choices B and C) implies damage to the ligaments that keep that joint tight. However, such damage would not produce the loud popping noise so characteristic of rupture of the Achilles tendon. Grating and crepitation (choice E) are findings that indicate bony fracture; if these were present, the patient would not be able to put weight on the injured side.

    Rate this question:

  • 19. 

    A 66-year-old man with diabetes and generalized arteriosclerotic occlusive disease notices a gradual loss of erectile function over several years. Initially, he can get erections, but they do not last long enough. Later, he notices a decrease in the quality of his erections, and more recently he becomes, by his own criteria, completely impotent. He has occasional, brief nocturnal erections, but "he can never get an erection when he needs one." Which of the following is the most appropriate initial step in management?

    • A.

      Psychotherapy

    • B.

      Pharmacologic therapy

    • C.

      Erectile nerve reconstruction

    • D.

      Implantable penile prosthesis

    • E.

      Pudendal artery revascularization

    Correct Answer
    B. Pharmacologic therapy
    Explanation
    This patient has organic impotence, but it is not related to trauma for which surgical reconstruction would be indicated. His remaining function can be augmented with sildenafil (Viagra®). Psychotherapy (choice A) is the thing to do for psychogenic impotence, which has a sudden onset rather than the gradual development described in this case. Nerve damage (as suggested in choice C) is the culprit in impotence following pelvic surgery (not the case here). As of now, there is no effective way to reanastomose those invisible little nerve fibers. Penile prosthesis (choice D) is always the last option, never the first one. Once a prosthesis is inserted, the normal erectile mechanism is destroyed forever. Had the history been that of a young man becoming impotent after a motorcycle accident, a vascular lesion would have been the likely problem, and a reconstruction (choice E) would be the thing to do.

    Rate this question:

  • 20. 

    A 54-year-old woman has a severe ureteral colic. An intravenous pyelogram shows a 7-mm ureteral stone at the ureteropelvic junction. She has a normal coagulation profile. Which of the following would most likely be the best therapy in this case?

    • A.

      Plenty of fluids and analgesics and await spontaneous passage

    • B.

      Extracorporeal shock wave lithotripsy (ESWL)

    • C.

      Endoscopic retrograde basket extraction

    • D.

      Endoscopic retrograde laser vaporization of the stone

    • E.

      Open surgical removal

    Correct Answer
    B. Extracorporeal shock wave lithotripsy (ESWL)
    Explanation
    Extracorporeal shock wave lithotripsy (ESWL) is the most commonly used method to fragment urinary stones and allow their passage. Pregnancy and coagulation problems are contraindications. The first one is ruled out by her age, the second one we have been told she does not have. Waiting for spontaneous passage (choice A) would have been perfect for a much smaller stone (3 mm) that had already negotiated most of the ureter. A 7-mm stone way up at the ureteropelvic junction has a very small chance of spontaneous passage. Retrograde endoscopic approaches (choices C and D) are more invasive than ESWL. They would not be the first choice for this scenario. Open surgical removal (choice E) would have been good for a much bigger stone. A huge target (a stone 3 cm or larger) could indeed be fragmented by ESWL, but then we would be contending with dozens of still very large stones. In those cases, a direct approach to extract the huge intact stone would work better.

    Rate this question:

  • 21. 

    A 68-year-old woman presents with an obviously incarcerated umbilical hernia. She has gross abdominal distention, is clinically dehydrated, and reports persistent fecaloid vomiting for the past 3 days. Although tired, weak, and thirsty, she is awake and alert and her sensorium is not particularly affected. Laboratory analysis reveals a serum sodium concentration of 118 mEq/L. Which of the following is the most likely physiologic explanation for the serum sodium?

    • A.

      She has acute water intoxication

    • B.

      She has been vomiting and trapping hypertonic fluids in the bowel lumen

    • C.

      She has vomited and sequestered sodium-containing fluids, and has retained endogenous and ingested water

    • D.

      There must be a laboratory error, because such a serum sodium level would have produced coma

    • E.

      Volume deprivation leads to renal wasting of sodium

    Correct Answer
    C. She has vomited and sequestered sodium-containing fluids, and has retained endogenous and ingested water
    Explanation
    Gastrointestinal tract fluids have a sodium concentration very close to that of plasma; as they are lost (internally or externally), they should be replaced with isotonic, sodium-containing fluids. But that is not what patients typically do at home. Thirsty and unable to eat solid (sodium-containing) foods, they drink water, Coke, and tea, fluids without significant amounts of sodium, which the body avidly retains because of the severe volume depletion. Endogenous water from catabolic activity is also retained. Dilutional hyponatremia eventually develops. She does not have "water intoxication" (choice A). This term denotes abnormal water retention due to excessive water infusion at a time when there is a high level of ADH in the blood. This patient is retaining water because she is desperately volume-depleted, not because high volumes of water are being forced into her. The hyponatremia is not due to the loss of hypertonic fluid (choice B). There are no hypertonic fluids in the gut, or anywhere else for that matter. The only hypertonic fluid that we can lose is highly concentrated urine, but we usually do so as a physiologic response to save water. Yes, we often see comatose and convulsing patients when they have this much hyponatremia (choice D), but that happens when water retention is massive and fast. Slow water retention allows the brain to adapt. One can see even lower serum sodium concentrations in patients with a clear sensorium. Volume deprivation leads to renal wasting of sodium (choice E) is plain wrong. What the kidney does when there is volume depletion is to increase reabsorption of sodium, not to dump it.

    Rate this question:

  • 22. 

    A 31-year-old woman smashes her car against a bridge abutment. She sustains multiple injuries, including upper and lower extremity fractures. She is fully awake and alert, and she reports that she was not wearing a seat belt and distinctly remembers hitting her abdomen against the steering wheel. Her blood pressure is 135/75 mm Hg, and her pulse is 88/min. Physical examination shows that she has a rigid, tender abdomen, with guarding and rebound in all four quadrants. She has no bowel sounds. Which of the following would be the most appropriate step in evaluating potential intraabdominal injuries?

    • A.

      Continued clinical observation

    • B.

      CT scan of the abdomen

    • C.

      Sonogram of the abdomen

    • D.

      Diagnostic peritoneal lavage

    • E.

      Exploratory laparotomy

    Correct Answer
    E. Exploratory laparotomy
    Explanation
    The presence of an "acute abdomen," which this woman has, is an indication for exploratory surgery and prompt repair of the injuries (probably affecting hollow viscera) that have produced the signs of peritoneal irritation. Continued clinical observation (choice A) would be irresponsible when it is clinically obvious that she already has an acute abdomen. What would one observe for? Development of septic shock? Death? CT scan (choice B) is ideal when the issue is potential intraabdominal bleeding in a hemodynamically stable patient who can be safely sent to the radiology department. CT scan might even be a good idea if the picture of acute abdomen were equivocal. But it is not needed here. Diagnostic peritoneal lavage (choice D) or sonogram done in the emergency department (choice C) are our options when we suspect intraabdominal bleeding and the patient is too unstable to be sent anywhere. As pointed out above, however, when an acute abdomen has clearly developed, it is time to operate.

    Rate this question:

  • 23. 

    A 27-year-old man sustained penetrating injuries of the chest and abdomen when he was repeatedly stabbed with a long ice-pick. At the time of admission, he had a right pneumothorax, for which a chest tube was placed prior to undergoing a general anesthetic for exploratory laparotomy. The operation revealed no intraabdominal injuries and was terminated sooner than had been anticipated. The patient remained intubated, waiting for the anesthetic to wear off. Because he was not moving enough air, he was placed on a respirator. Then, he suddenly went into cardiac arrest and died. All through this time he had been hemodynamically stable, and never had any signs of hypotension or arrhythmias. Which of the following was the most likely cause of the cardiac arrest?

    • A.

      Air embolism

    • B.

      Fat embolism

    • C.

      Myocardial infarction

    • D.

      Pulmonary embolus

    • E.

      Tension pneumothorax

    Correct Answer
    A. Air embolism
    Explanation
    Truly sudden death, with no warnings whatsoever, brings to mind the possibility of air embolism. The mechanism in this case is suggested by the circumstances. The patient had deep penetrating injuries that may have involved a major vein and an adjacent bronchus. When he was placed on the respirator, the air was forced through from the tracheobronchial tree into the vein, and thus into the heart. Fat embolism (choice B) is seen with multiple long bone fractures, and the symptomatology is respiratory failure. Myocardial infarction (choice C) would be extremely unlikely in a young man who was never hypotensive, and never showed arrhythmias. Pulmonary embolus (choice D) is seen late in the postoperative period after several days of reduced mobility. This man would have had no opportunity to develop clots in major veins in such short clinical course. Tension pneumothorax (choice E) would be unlikely to develop with a chest tube in place. However, even if we assume the tube was clogged or kinked and thus not functioning properly, a tension pneumothorax does not cause sudden death: it causes progressive hemodynamic shock and respiratory distress.

    Rate this question:

  • 24. 

    A 13-year-old, obese boy complains of persistent knee pain for several weeks. The family brings him in because he has been limping. He sits in the examining table with the sole of the foot on the affected side pointing to the other leg. Physical examination is normal for the knee, but shows limited hip motion. As the hip is flexed, the leg goes into external rotation and cannot be rotated internally. Which of the following is the most likely diagnosis? 

    • A.

      Avascular necrosis of the femoral head

    • B.

      Developmental dysplasia of the hip

    • C.

      Osteogenic sarcoma of the lower femur

    • D.

      Slipped capital femoral epiphysis

    • E.

      Tibial torsion with foot inversion

    Correct Answer
    D. Slipped capital femoral epiphysis
    Explanation
    Slipped capital femoral epiphysis is an orthopedic emergency. The clinical picture is classic: a chubby male in his early teens who is limping and cannot rotate his leg internally. Part of the classic presentation is also the fact that often hip pathology produces knee pain, but the knee is normal on physical examination. Avascular necrosis (choice A) is seen in younger children, around the age of 6 years. Developmental dysplasia (choice B) is typically diagnosed at birth. If it is missed, the affected child develops significant sequelae early in life. Osteogenic sarcoma (choice C) happens in the late teens, but the clinical findings are different, with a bony mass at the area of tenderness and no limitation on the motion of the hip. Tibial torsion and foot inversion (choice E) would not limit hip motion

    Rate this question:

  • 25. 

    A 72-year-old man has a 3-mm ureteral stone impacted at the ureterovesical junction. He has been having mild ureteral colicky pain for about 12 hours, and he has been given fluids and analgesics in the expectation that he will spontaneously pass the stone. He then has shaking chills, and spikes a temperature of 40 C (104 F). When seen shortly thereafter, he has flank pain and looks quite ill. Which of the following is the most appropriate next step in management?

    • A.

      Addition of IV antibiotics to the current therapeutic regimen

    • B.

      Crushing and extraction of the stone via cystoscopy

    • C.

      Extracorporeal shock wave lithotripsy and parenteral antibiotics

    • D.

      Immediate insertion of a suprapubic catheter into the bladder

    • E.

      IV antibiotics and immediate decompression of the urinary tract above the stone

    Correct Answer
    E. IV antibiotics and immediate decompression of the urinary tract above the stone
    Explanation
    The combination of obstruction and infection in the urinary tract constitutes a dire emergency that requires, in addition to IV antibiotics, the immediate decompression of the urinary tract above the point of obstruction. Adding antibiotics without decompressing the urinary tract (choice A) is not enough. Rapid destruction of the kidney, and even death from septic shock, will ensue if decompression is not done. It is too late to crush and remove the stone (choice B) once the infection has occurred. Complicated instrumentation should not be done in these circumstances. Such steps should await resolution of the lethal infection-obstruction combination. The same can be said for the use of extracorporeal shock wave lithotripsy (choice C). If it is chosen as the way to manage the stone, it should be done when infection and obstruction are no longer present. Putting a catheter into the bladder (choice D) would provide decompression below the level of obstruction. The drainage of infected urine is needed above the obstructing point.

    Rate this question:

  • 26. 

    A 14-year-old boy dives into the shallow end of a swimming pool and hits his head against the bottom. When he is rescued, he shows a complete lack of neurologic function below the neck. He is still breathing on his own, but he cannot move or feel his arms and legs. The paramedics carefully immobilize his neck for transportation to the hospital, and they alert the emergency department to his impending arrival. Once there, which of the following would most likely have an immediate benefit for this patient? 

    • A.

      Hyperbaric oxygenation

    • B.

      IV antibiotics

    • C.

      IV high-dose corticosteroids

    • D.

      Massive diuresis induced by loop diuretics.

    • E.

      Surgical decompression of the cord

    Correct Answer
    C. IV high-dose corticosteroids
    Explanation
    . There is some evidence that high-dose corticosteroids administered as soon as possible after the injury will result in a better ultimate outcome. Although the true medical value of this observation may be debatable, there is a legal imperative to use the treatment, which offers some hope and has not been shown to be detrimental. Hyperbaric oxygenation (choice A) has no role in the acute management of neurologic injuries. Antibiotics (choice B) are likewise unlikely to affect the course of events in a case like this. Although diuresis (choice D) is part of the therapy used to decrease intracranial pressure, the agent of choice is mannitol, and the indications do not include spinal cord injury. Surgical decompression (choice E) might be done, but the decision is individualized depending on the findings on MRI. Not all patients are automatically and immediately taken to the operating room.

    Rate this question:

  • 27. 

    A 72-year-old man is scheduled to have elective sigmoid resection for diverticular disease. He has a history of heart disease, and had a documented myocardial infarction 2 years ago. He currently does not have angina, but he lives a sedentary life because "he gets out of breath" if he exerts himself. During the physical examination, it is noted that he has jugular venous distention. He has a hemoglobin of 12 g/dL. If surgery is indeed needed, which of the following should most likely be done prior to the operation?

    • A.

      Evaluate the patient as a candidate for coronary revascularization

    • B.

      Place the patient on intensive respiratory therapy

    • C.

      Order a transfusion to increase the patient's hemoglobin to 14 g/dL

    • D.

      Treat the patient for congestive heart failure

    • E.

      If at all possible, wait 6 months before performing surgery

    Correct Answer
    D. Treat the patient for congestive heart failure
    Explanation
    Jugular venous distention in this setting is indicative of congestive heart failure, a condition that would make elective surgery very risky. Medical treatment for congestive heart failure can reduce the risk. Coronary revascularization (choice A) is a preoperative consideration for patients with progressive angina. Preoperative respiratory therapy (choice B) is needed for patients with chronic obstructive pulmonary disease (COPD) who have compromised ventilation. A hemoglobin of 12 g/dL should be sufficient, and bringing it up to 14 g/dL (choice C) by transfusions would aggravate the existing congestive failure. Waiting 6 months (choice E) is imperative before noncardiac surgery is done after a myocardial infarction. In this case, the infarction happened 2 years ago; thus, the waiting period has already taken place.

    Rate this question:

  • 28. 

    A group of illegal immigrants is smuggled across the border in a closed metal truck in the middle of summer. When apprised by radio that the border patrol is on their trail, the smugglers abandon their charges in the middle of the desert, in the locked truck, with little water to drink. The victims are found and rescued 5 days later. One of them is brought to the emergency department, awake and alert, with obvious clinical signs of severe dehydration and a serum sodium concentration of 155 mEq/L. Which of the following would be the best choice and rate of IV fluid administration?

    • A.

      5 L of 5% dextrose in water (D5W) over 2-3 days

    • B.

      5 L of D5W over 5-10 hours

    • C.

      5 L of 5% dextrose in half normal saline (D5 1/2 NS) over 5-10 hours

    • D.

      10 L of D5 1/2 NS over 5-10 hours

    • E.

      10 L of normal saline over 2-3 days

    Correct Answer
    C. 5 L of 5% dextrose in half normal saline (D5 1/2 NS) over 5-10 hours
    Explanation
    A rough guideline to quantify water loss is that every 3 mEq/L that the serum sodium concentration is above normal, represents about 1 L of water deficit. With a value of 155, we can assume a water deficit of about 5 L. There is no advantage to the patient in remaining severely volume contracted for several days, thus the replacement should aim for correction in a matter of 5-10 hours rather than 2 or 3 days. However, because his loses were incurred slowly (over 5 days), his brain has had a chance to adapt to the tonicity change (he is indeed awake and alert). Thus, the tonicity correction should not happen with the same speed with which the volume is going to be corrected. That delay is achieved by choosing a fluid that is not pure water, but one that has some sodium in it to dampen the effect on tonicity. Half normal saline is a good choice. 5 L of D5W over 2 or 3 days (choice A) would be safe from the viewpoint of slowly correcting the tonicity, but it would unnecessarily prolong the state of volume depletion. 5 L of D5W over 5-10 hours (choice B) could well be deadly, because it would revert the tonicity to normal at a rate too fast for the brain to follow. Choices D and E budget a volume replacement well beyond what is needed. Neither would be lethal, because D5W is not used, but neither of them is the best answer.

    Rate this question:

  • 29. 

    A 72-year-old man comes in complaining of persistent and nagging low back pain that he has had for several weeks. The pain seems to be increasing in intensity, is worse at night, is unrelieved by rest or positional changes, and is not exacerbated by coughing, sneezing, or straining to have a bowel movement. He is a chronic smoker, and for the past 3 months has had persistent cough with occasional bloody streaked sputum, as well as a 20-pound weight loss. On physical examination, he is distinctly tender to palpation at a particular spot over his lower thoracic spine. Which of the following is the most likely diagnosis?

    • A.

      Ankylosing spondylitis

    • B.

      Herniated disk

    • C.

      Metastatic tumor to the thoracic spine

    • D.

      Multiple myeloma

    • E.

      Primary malignant bone tumor

    Correct Answer
    C. Metastatic tumor to the thoracic spine
    Explanation
    The age, nature of the pain, physical findings, and associated symptoms are all highly suggestive of metastatic tumor, and the source is probably the lung. Ankylosing spondylitis (choice A) happens to younger patients (in their early 30s) who have pain and stiffness in the mornings, and relief as they become active during the day. Herniated disc (choice B) can virtually be excluded by the fact that the pain is not exacerbated by coughing, sneezing, or straining. Multiple myeloma (choice D) is also a disease of old men, but they get anemia and multiple lytic lesions throughout the skeleton. Primary malignant bone tumors (choice E) occur in much younger people, and the extremities are a more likely location.

    Rate this question:

  • 30. 

    The unrestrained front-seat passenger in a car that crashes at high speed is brought to the emergency department by paramedics. At the site of injury, the patient was unconscious and had gurgly respiratory sounds, and the EMTs successfully accomplished blind nasotracheal intubation. The initial survey in the emergency department shows normal vital signs, multiple facial lacerations, and an unresponsive, comatose patient with fixed dilated pupils. Preparations are made to do a CT scan of the head. It is imperative that which of the following should be obtained as well?

    • A.

      Base of the skull x-ray films

    • B.

      Extension of the CT to include the entire cervical spine

    • C.

      Radiographs of all the teeth

    • D.

      Separate CT scan of the abdomen

    • E.

      Special views of the maxillary sinuses

    Correct Answer
    B. Extension of the CT to include the entire cervical spine
    Explanation
    Deceleration injury resulting in head trauma of sufficient magnitude to produce coma and multiple facial lacerations may very well have also produced injuries to the cervical spine. Although intubation in the field was very appropriate before the cervical spine was visualized (the patient was comatose and had signs of compromised airway), we need to know the status of the cervical spine before much more is done to the patient. Since CT is needed to evaluate his head injury, the most expeditious way to check his cervical spine is by extending the CT to include the neck. The base of the skull (choice A) will be very well visualized in the CT. No separate x-ray films are needed. Checking the status of the teeth (choice C) or the sinuses (choice E) are hardly the sort of emergencies that need to be addressed now. A separate CT scan of the abdomen (choice D) would have been indicated if he had been hypotensive. So far we have no indication of internal bleeding, and thus do not need to look for a source.

    Rate this question:

  • 31. 

    A 55-year-old woman has been known for years to have mitral valve prolapse. She has now developed exertional dyspnea, orthopnea, and atrial fibrillation. She has an apical, high-pitched, holosystolic heart murmur that radiates to the axilla and back. Because of her deterioration, surgery has been recommended. Which of the following is the most appropriate procedure?

    • A.

      Aortic valve replacement

    • B.

      Mitral commissurotomy

    • C.

      Mitral valve annuloplasty

    • D.

      Mitral valve replacement

    • E.

      Both aortic and mitral valve replacement

    Correct Answer
    C. Mitral valve annuloplasty
    Explanation
    The physical findings are classic for mitral valve insufficiency. Whenever possible, repair of the native mitral valve is preferable to replacement. The way to repair an insufficient valve is to tighten the annulus, bringing the leaflets closer to one another. There are no physical findings indicating involvement of the aortic valve; therefore, choices A and E are wrong. Mitral commissurotomy (choice B) is the preferred operation for mitral stenosis. This patient has no signs of stenosis, and no history of rheumatic heart disease to suggest that she might have it. Mitral valve replacement (choice D) is the choice when repair of the native valve cannot be done.

    Rate this question:

  • 32. 

    A 23-year-old woman seeks help for exquisite pain with defecation and blood streaks on the outside of her stools, which she has been having for several weeks. Because of the pain, she has avoided having bowel movements, and when she finally did the stools were hard and even more painful. When seen, she has no fever or leukocytosis. Physical examination has to be done under spinal anesthesia, because the patient was so afraid of the pain that she initially refused even inspection of the area. The examination confirms the suspected diagnosis, and she is placed on stool softeners and appropriate topical agents, but without success. She is willing to undergo more aggressive treatment. Which of the following is the most appropriate next step?

    • A.

      Excision of the lesion

    • B.

      Fistulotomy

    • C.

      Incision and drainage

    • D.

      Lateral internal sphincterotomy

    • E.

      Rubber band ligation

    Correct Answer
    D. Lateral internal sphincterotomy
    Explanation
    The clinical picture is classic for anal fissure, which is perpetuated by the fact that the anal sphincter is "too tight." Forceful dilatation under anesthesia, lateral sphincterotomy, or botulinum toxin injections are acceptable options to "break the cycle." The only one of those choices given is the sphincterotomy. Excision (choice A) used to be done for this condition, before the role of the "too tight sphincter" was elucidated. Fistulotomy (choice B) is not the answer. She has a fissure, not a fistula. Incision and drainage (choice C) is another option that addresses a wrong diagnosis. We do that for perirectal abscess, which produces severe pain with fever and leukocytosis, but without blood streaks, and drains spontaneously after several days if not diagnosed and treated. Rubber band ligation (choice E) is the answer for internal hemorrhoids. Internal hemorrhoids can bleed, but typically do not hurt. Thrombosed external hemorrhoids can hurt tremendously, but those are not amenable to rubber band ligation.

    Rate this question:

  • 33. 

    An exploratory laparotomy for multiple intra-abdominal injuries has lasted 3 and a half hours. Multiple blood transfusions have been given, and several liters of Ringer's lactate have been infused. When the surgeons are ready to close the abdomen, they find that the abdominal wall edges cannot be pulled together without undue tension. Both the belly wall and the abdominal contents seem to be swollen. Which of the following is the most appropriate management in this situation? 

    • A.

      Approximate the skin only, using towel clips

    • B.

      Close the abdomen with heavy retention sutures

    • C.

      Give diuretics and close the abdomen in the usual way

    • D.

      Leave the abdomen and its contents open to the air

    • E.

      Provide temporary bowel coverage with an absorbable mesh

    Correct Answer
    E. Provide temporary bowel coverage with an absorbable mesh
    Explanation
    This is a new entity known as the abdominal compartment syndrome, unknown in the days when severely traumatized patients died prior to, or during, surgery. The life-saving massive fluid infusions produce severe edema in the surgical field. Forced closure would compromise ventilation and venous return. A temporary plastic coverage, or a mesh, allows the bowel to be protected without undue pressure. Closing the skin only (choice A) can be life-saving when hypothermia develops during surgery. In this setting, however, the skin will not come together without undue tension. Forced closure (choice B) would compromise ventilation and produce acute renal failure due to pressure on the inferior vena cava. Diuretics (choice C) cannot selectively remove the fluid from the swollen tissues. Leaving the bowel exposed to the air (choice D) is not an option. In the short term, the patient would suffer severe heat loss; later, the bowel would dry out and perforate.

    Rate this question:

  • 34. 

    A 61-year-old man comes in because of colicky abdominal pain and vomiting of 3 days' duration. On physical examination, he is moderately distended and has high pitched hyperactive bowel sounds and a 5-cm tender groin mass. On direct questioning, he explains that he has had that bulge for many years, but has always been able to "push it back in" when he lies down. For the past 3 days, however, he has been unable to do so. He has a temperature of 38.9 C (102 F) and a white blood cell count of 12,500/mm3. Which of the following is the most appropriate management at this time?

    • A.

      A sonogram of the mass

    • B.

      A trial of nasogastric suction and IV fluids for a few days

    • C.

      Insertion of a long rectal tube via sigmoidoscopy

    • D.

      Manual reduction of the hernia, followed by a period of observation

    • E.

      Urgent surgical intervention

    Correct Answer
    E. Urgent surgical intervention
    Explanation
    The clinical picture is that of a strangulated inguinal hernia. If he only had the tender mass without signs of intestinal obstruction, he might have omentum trapped. If he had the intestinal obstruction without fever, leukocytosis, and the tender mass, he could be obstructed but not strangulated. But, the combination that he has is clearly that of obstruction with strangulation. He needs urgent surgery. A sonogram to make a diagnosis (choice A) might be appropriate for a mass without signs of obstruction, if we could not clinically be sure that it was a hernia. Nasogastric suction and IV fluids (choice B) is the standard approach for obstruction due to adhesions, when there are no signs suggestive of strangulation. We do not operate for adhesions (they form again), but do so only to rescue the bowel that is trapped. In hernias, on the other hand, we want not only to rescue the bowel but also to repair the hernia. A long rectal tube (choice C) is used in Ogilvie's syndrome or volvulus, but not in strangulated hernias. Manual reduction (choice D) would actually be dangerous in this case, as it might force a dead segment of bowel into the abdomen, increasing morbidity and delaying definitive treatment. If he had no fever, no leukocytosis, and no tenderness, such an approach might be justified to gain time for an elective, non-rushed hernia repair.

    Rate this question:

  • 35. 

    In a rollover car accident, a 42-year-old woman is thrown from the car. The car subsequently lands on her and crushes her. On physical examination in the emergency department, it is determined that she has a pelvic fracture, which is confirmed by portable x-rays done as she is being resuscitated. Her initial blood pressure is 50/30 mm Hg, and her pulse is 160/min and barely perceptible. Thirty minutes later, after 2 L Ringer's lactate and 2 U packed cells have been infused, her pressure is only 70/50 mm Hg, and her pulse is 130/min. A sonogram done in the emergency department shows no intra-abdominal bleeding, and a diagnostic peritoneal lavage confirms that there is no blood in the abdomen (the recovered fluid is pink, but not grossly bloody). Rectal and vaginal exams show no injuries to those organs. There is no blood in her urine. Which of the following is the most appropriate next step in management?

    • A.

      Packing of the vagina and rectum

    • B.

      Angiographic embolization of torn veins

    • C.

      External fixation of the pelvis

    • D.

      Open reduction and internal fixation of the pelvis

    • E.

      Exploratory laparotomy with pelvic dissection and hemostasis

    Correct Answer
    C. External fixation of the pelvis
    Explanation
    This is actually a terrible situation, with no easy way out. Pelvic fractures can bleed massively, and often the source is torn veins that are not easily controlled. Minimizing the motion of the bone fragments by external fixation can be helpful, and it will not make the situation worse. Packing the vagina or rectum (choice A) would help if bleeding originated in those organs, but they cannot reach the source of bleeding in this case. Angiography (choice B) can be very helpful when arteries are torn. It cannot do the same for veins. Opening the fractured area (choice D) would lose the tamponade effect and would not help control the bleeding. And as for the surgeons coming to the rescue (choice E), this is one place in which the high and mighty are routinely humbled. Opening the pelvic hematoma loses the tamponade effect, and once into the thick of things, pelvic veins bleed massively and are not easily controlled. It is best to stay out of these situations.

    Rate this question:

  • 36. 

    A 52-year-old nurse seeks medical retirement because of a "heart condition." She complains of disabling attacks of tachycardia and palpitations. The physical examination and ECG studies confirm that indeed her pulse is between 100 and 105/min at all times, and she is in and out of atrial fibrillation. It is also noted that she is fidgety and constantly moving, and various examiners remark that she arrives for tests lightly dressed when it is rather cold outside. Thyroid function studies show elevated free thyroxine (T4) and undetectable levels of thyroid stimulating hormone (TSH). Her thyroid gland is not clinically enlarged or tender. Which of the following is the most appropriate next step in diagnosis?

    • A.

      Fine needle aspiration cytology of the thyroid gland

    • B.

      MRI of the pituitary area

    • C.

      Radioactive iodine uptake

    • D.

      Serum levels of C peptide

    • E.

      Serum levels of triiodothyronine (T3)

    Correct Answer
    C. Radioactive iodine uptake
    Explanation
    It has been established clinically and by the laboratory that this patient is hyperthyroid, but the thyroid gland does not seem to be abnormal. The circumstances suggest that self-administration of thyroid hormone for secondary gain (e.g., weight loss) is a possibility. Radioactive iodine uptake should be high if her gland is indeed hyperfunctioning, but it will be near zero if it is suppressed by the exogenous hormone. Malignancy is not an issue; thus, fine needle aspiration (choice A) does not have a role. High thyroid function with undetectable levels of TSH excludes the pituitary as the source of the problem. Thus, there is no reason to investigate it as suggested in choice
    B. C peptide (choice D) is indeed used to ferret out hormonal self-administration, but it distinguishes endogenous from exogenous insulin, not thyroid hormone. T3 (choice E) needs to be determined when clinical signs of hyperthyroidism coexist with normal levels of T4.

    Rate this question:

  • 37. 

    A 59-year-old man is referred for evaluation because he has been fainting at his job, where he operates heavy machinery. He is pale and gaunt, but otherwise his physical examination is remarkable only for 4+ occult blood in the stool. Laboratory studies show a hemoglobin of 5 gm/dL with microcytosis, as well as decreased levels of serum iron and increased iron binding capacity. Which of the following will most likely establish the diagnosis? 

    • A.

      Upper gastrointestinal series (swallowed barium studies)

    • B.

      Colonoscopy

    • C.

      Flexible sigmoidoscopy to 45 cm

    • D.

      Upper gastrointestinal endoscopy

    • E.

      Visceral angiogram

    Correct Answer
    B. Colonoscopy
    Explanation
    Iron deficiency anemia in the adult is always due to chronic blood loss, and the source is obvious in this vignette: the gastrointestinal tract. In turn, the most likely site, in the absence of other symptoms, is a cancer of the right side of the colon, which is best seen by colonoscopy. Upper gastrointestinal series (choice A) would not be likely to reveal the source of this man's anemia, since the cecum or ascending colon is the number one target. Flexible sigmoidoscopy (choice C) would not reach the likely site of the cancer. If the cancer were located in the left colon, he would likely have visible blood in his stools and a change in bowel habits. Upper gastrointestinal endoscopy (choice D) is the first test when someone vomits blood. It often will also reveal the source of occult blood loss (peptic ulcer disease or aspirin-related gastritis) when the colon is found to be normal. In this case, however, the cecum or ascending colon is the number one target. As for visceral angiogram (choice E), it would be great at the time of massive gastrointestinal bleeding (more than 2 mL/min), but in this example it would be a very expensive, invasive, and roundabout way to demonstrate the presence of a tumor (by tumor blush).

    Rate this question:

  • 38. 

    A 24-year-old woman is admitted to the hospital for a broken femur. The patient was in a motor vehicle accident 20 hours ago and was brought to the hospital by EMS. On the scene, she was found belted in her car in the drivers seat, and her only documented injury was the leg fracture. She had no loss of consciousness or altered mental status. On arrival to the hospital, radiographs confirmed a fracture of her femur. She was stabilized over night and scheduled for surgery the next day. Which of the following is the major surgical risk for this patient?

    • A.

      Air embolism

    • B.

      Cerebrovascular accident

    • C.

      Fat embolism

    • D.

      Osteomyelitis

    • E.

      Permanent disability

    Correct Answer
    C. Fat embolism
    Explanation
    If a transesophageal echo probe is placed in every patient undergoing femoral reaming for fracture repair, the incidence of fat and particle debris in the right atrium approaches 70%. In fact, a major risk of lower extremity orthopedic procedures is pulmonary embolism due to fat or clots. The intramedullary pressures generated during the repair are greater than 500 psi and are enough to cause venous extrusion of fat and other particulate matter into the circulation. Air embolism (choice A) is common with neurosurgical procedures but is not often seen with orthopedic procedures of the lower extremity. Although cerebrovascular accident (choice B) can occur in the absence of a patent foramen ovale, it is rare. Osteomyelitis (choice D) is a feared complication of orthopedic surgery, and precautions such as sterile preparations and antibiotics are taken to guard against it. Because of this, the complication of pulmonary embolism due to fat is much greater than that of bone infection. Permanent disability (choice E), although a vague term, would rarely be the result of a lower extremity fracture repair.

    Rate this question:

  • 39. 

    A 42-year-old woman hit her breast with a broom handle while doing housework. She noticed a lump in that area at the time, and 1 week later the lump was still present. She then sought medical advice. On physical examination, she has a 3-cm, hard mass deep inside the affected breast, and some superficial ecchymosis over the area. Which of the following is the most appropriate next step, or steps, in management?

    • A.

      Reassess in about 2 months, with no specific therapy

    • B.

      Hot packs, analgesics, and surgical evacuation of the hematoma

    • C.

      Mammogram, and no further therapy if the report does not identify cancer

    • D.

      Mammogram and biopsy of the mass

    • E.

      Mastectomy

    Correct Answer
    D. Mammogram and biopsy of the mass
    Explanation
    Although the history of trauma might suggest a hematoma or fat necrosis, it is well known that trivial trauma sometimes brings to the attention of the patient an area of the body that had not been examined before. A breast mass in a 42-year-old woman requires a mammogram and biopsy. Waiting 2 months (choice A) would be unacceptable for a potential cancer. Hot packs and analgesics (choice B) on the assumption that this is a hematoma would also delay the diagnosis if a cancer is present. Furthermore, if this is indeed a hematoma one would not necessarily want to drain it. Choice C is incorrect because the mammogram is an adjunct to the biopsy of a breast mass, not a substitute for it. The two studies are complementary. Mastectomy (choice E) is too radical a step before a diagnosis has been established.

    Rate this question:

  • 40. 

    The unrestrained front-seat passenger in a car that crashed at high speed arrives at the emergency department with signs of moderate respiratory distress. Physical examination shows no breath sounds at all on the left hemithorax. Percussion is unremarkable, and his vital signs are normal. A chest x-ray film shows a collapsed left lung and multiple air-fluid levels filling the left pleural cavity. A nasogastric tube that had been placed prior to taking the film shows the tube reaching the upper abdomen and then curling up into the left chest. Which of the following is the most likely diagnosis?

    • A.

      Blow out of pulmonary blebs

    • B.

      Esophageal rupture or perforation

    • C.

      Left diaphragmatic rupture

    • D.

      Left hemopneumothorax

    • E.

      Major injury to the tracheobronchial tree

    Correct Answer
    C. Left diaphragmatic rupture
    Explanation
    The left diaphragm can blow out with blunt injuries, allowing the bowel to move up into the chest. The multiple air-fluid levels suggest that bowel is indeed there, and the trajectory of the nasogastric tube confirms that the abdominal viscera (including the stomach) have been sucked up into the thoracic cavity. Pulmonary blebs (choice A) produce a pneumothorax when they rupture. The esophagus (choice B) virtually never ruptures with blunt abdominal trauma. You need a penetrating injury, or better yet an endoscopy, to perforate it. When that happens, the outcome is mediastinitis. A hemopneumothorax (choice D) can indeed happen in thoracic injuries, but the x-ray films would show one single large air-fluid level, and the nasogastric tube would be in the stomach, without curling up into the chest. The tracheobronchial tree (choice E) can indeed break as a consequence of deceleration injuries, but the outcome would be a pneumothorax and air in the mediastinum and the subcutaneous tissues.

    Rate this question:

  • 41. 

    On the 7th postoperative day after the pinning of a broken hip, a 76-year-old man suddenly develops severe pleuritic chest pain and shortness of breath. When examined, he is found to be anxious, diaphoretic, and tachycardic, with a blood pressure of 140/85 mm Hg. He has prominent distended veins in his neck and forehead. Blood gases show hypoxemia and hypocapnia. His chest x-ray film is unremarkable. The nurses have placed him on supplemental oxygen by face mask. Which of the following is the most appropriate next step in management? 

    • A.

      Aortogram and emergency surgical repair

    • B.

      ECG and cardiac enzymes

    • C.

      Intubation and respirator, with hyperventilation and PEEP

    • D.

      Retinal examination looking for fat droplets

    • E.

      Ventilation-perfusion lung scan, or spiral CT scan of the chest

    Correct Answer
    E. Ventilation-perfusion lung scan, or spiral CT scan of the chest
    Explanation
    The clinical picture is that of a pulmonary embolus. Although pulmonary angiogram is the "gold standard" diagnostic test, confirmation is usually obtained with the less invasive ventilation-perfusion scan. In some centers, the high-definition spiral CT scan has been found to be a better diagnostic modality, and it is noninvasive. Aortogram and surgical repair (choice A) assumes the pain is due to a dissecting aortic aneurysm. Had that been the case, the pain would have been more likely to radiate down toward the back, and the patient would have been severely hypertensive. ECG and cardiac enzymes (choice B) would probably be done in this patient, but only to rule out myocardial infarction with greater certainty. Clinically, we should be suspecting a pulmonary embolus, and negative ECG and negative enzymes would not specifically confirm the suspected diagnosis. Hyperventilation (choice C) is not needed on someone who already has hypocapnia. Looking for fat droplets (choice D) is not particularly useful, even when the clinical diagnosis suggests fat embolism. Fat embolism is more likely to be seen with multiple comminuted fractures of long bones (not just a broken hip), and the clinical manifestations are those of respiratory failure. There would be no chest pain.

    Rate this question:

  • 42. 

    A young man sustains a gunshot wound to the base of his neck. He was shot point blank with a .38 caliber revolver. The entrance wound is above the left clavicle, below the level of the cricoid cartilage, and just lateral to the sternomastoid muscle. The exit wound is just above the spinous process of the right scapula. He has normal breath sounds on both sides, is awake and alert, is talking with a normal tone of voice, is neurologically intact, and is hemodynamically stable. Portable x-ray films of the neck and chest taken in the emergency department show some air in the tissues of the lower neck, but are otherwise non-diagnostic. Which of the following is the most appropriate next step in management?

    • A.

      Observation for several hours

    • B.

      CT scan of the lower neck and upper chest

    • C.

      Angiogram, esophagogram, esophagoscopy, and bronchoscopy prior to surgical exploration

    • D.

      Immediate surgical exploration of the lower neck through a collar incision

    • E.

      Immediate surgical exploration of the upper chest through a median sternotomy

    Correct Answer
    C. Angiogram, esophagogram, esophagoscopy, and bronchoscopy prior to surgical exploration
    Explanation
    Gunshot wounds to the base of the neck need exploratory surgery, but the exact approach and incision are determined by a more accurate knowledge of the location and extent of the injuries. Thus, if time permits, diagnostic studies should precede surgical intervention. The major vessels, the tracheobronchial tree, and the esophagus are the potential targets that have to be investigated. Observation (choice A) might be appropriate for a stab wound in a completely asymptomatic patient. In gunshot wounds, we have to expect that injuries will exist, and they should not be neglected waiting for overt clinical signs. CT scan (choice B) has done wonders for our assessment of closed head injuries and blunt abdominal trauma, but it is not the study that would tell us what has happened to the major vessels, the esophagus, or the tracheobronchial tree in a gunshot wound. Immediate surgical exploration, either through the neck or the chest, or in combination, might be forced by a rapidly deteriorating situation. In the absence of such imperative, a decision to open the neck (choice D) or the chest (choice E) is premature at this point

    Rate this question:

  • 43. 

    A 67-year-old man shows up in the emergency department because he has not been able to void for the past 12 hours. He feels the need to, but he cannot do it. He gives a history that, for several years now, he has been getting up four or five times a night to urinate. It would take him a considerable time to get the urinary stream going, and the stream lacked force and often ended in a dribble. Because of a cold, 2 days ago he began taking an antihistamine, taking a decongestant, and drinking plenty of fluids. Physical examination shows a palpable, smooth, round mass arising from the pubis and reaching about half way toward the umbilicus. The mass is dull to percussion, and pushing on it accentuates the feeling of needing to void. Rectal examination reveals a large, boggy, non-tender prostate gland without nodules. This a classic presentation for which of the following acute conditions? 

    • A.

      Bacterial prostatitis

    • B.

      Cystitis in a patient with bladder cancer

    • C.

      Renal failure

    • D.

      Urinary retention in a patient with benign prostatic hypertrophy

    • E.

      Urinary retention in a patient with prostatic cancer

    Correct Answer
    D. Urinary retention in a patient with benign prostatic hypertrophy
    Explanation
    The history and rectal examination findings are classic for benign prostatic hypertrophy. The use of a decongestant has led to stimulation of alpha adrenergic receptors, which have further closed the bladder neck. A big, palpable bladder has resulted. Bacterial prostatitis (choice A) would produce pain, fever, and a very tender prostate on rectal examination. Cystitis and bladder cancer (choice B) could be expected to produce irritative symptoms and hematuria, but not painless retention. Renal failure (choice C) produces oliguria, not anuria. The bladder would be empty and thus not palpable. The patient would urinate small amounts and feel no need to urinate more. Prostatic cancer (choice E) is usually first felt as a stony hard nodule. It would be unusual for it to grow to a size at which complete obstruction is the first manifestation. If it did, though, the prostate would feel stony hard.

    Rate this question:

  • 44. 

    Several months after sustaining a crushing injury to his arm, a patient complains bitterly about constant, burning, agonizing pain in that arm, that does not respond to the usual analgesic medications. The pain in his arm is aggravated by the slightest stimulation of the area, such as rubbing from the shirt sleeves. The arm is cold, cyanotic, and moist, but it is not swollen. Pulses at the wrist are normal, and neurologic function of the three major nerves is intact. Which of the following is most appropriate to provide diagnostic confirmation of the nature of the problem and eventual therapy?

    • A.

      Angiogram and subclavian vein bypass

    • B.

      Cervical spine x-rays and cervical rib resection

    • C.

      Doppler studies and arterial reconstruction

    • D.

      Doppler studies and fasciotomy

    • E.

      Sympathetic block and surgical sympathectomy

    Correct Answer
    E. Sympathetic block and surgical sympathectomy
    Explanation
    The description is that of causalgia, also known as reflex sympathetic dystrophy. If sympathetic block relieves the symptoms, permanent cure will be obtained with surgical sympathectomy. Venous occlusion (choice A) would produce swelling but not this kind of pain. Cervical ribs (choice B) can produce neurologic and vascular symptoms in the arm, but they are related to activity and position and do not have the nature described here. Normal pulses make arterial insufficiency (choice C) unlikely. Furthermore, there is no description of intermittent claudication. Compartment syndrome (choice D) might have happened at the time of injury, but if that were the case, it would be too late to do a fasciotomy.

    Rate this question:

  • 45. 

    A 57-year-old alcoholic man is being treated for acute hemorrhagic pancreatitis. He was in the intensive care unit for 1 week, where he required chest tubes for pleural effusions and was on a respirator for several days. Eventually, he improved sufficiently to be transferred to the floor. Three days after leaving the unit, and about 2 weeks after the onset of the disease, he spikes a fever and develops leukocytosis. Which of the following developments do these recent findings most likely suggest?

    • A.

      Chronic pancreatitis

    • B.

      Pancreatic abscess

    • C.

      Pancreatic pseudocyst

    • D.

      Pelvic abscess

    • E.

      Subphrenic abscess

    Correct Answer
    B. Pancreatic abscess
    Explanation
    A very common complication of hemorrhagic pancreatitis, and often the reason for the demise of the patient, is the development of a pancreatic abscess. The timetable is usually about 10-14 days from the onset of the disease, and the initial manifestations are fever and leukocytosis. Chronic pancreatitis (choice A) develops after several years of recurrent attacks of pancreatitis, and is characterized by steatorrhea, diabetes, and constant pain. Pancreatic pseudocyst (choice C) is another potential complication of pancreatitis, but the manifestations are related to pressure symptoms from the fluid collection, there is no fever or leukocytosis, and the timetable for development is about 6 weeks from the onset of the disease. Pelvic abscess (choice D) and subphrenic abscess (choice E) are indeed in the differential diagnosis, as they also show up with fever and leukocytosis some 10-14 days from the original problem. But, the original problem for these patients is usually an infectious process in the abdomen, e.g., a ruptured appendix or a perforated viscus. If the problem began with pancreatitis, and then there are signs of sepsis, the pancreas is the logical place to harbor the pus.

    Rate this question:

  • 46. 

    A 31-year-old male immigrant from India is found on a routine physical examination to have a single, 2-cm nodule in the right lobe of his thyroid gland. The mass is firm, moves up and down with swallowing, and is not tender. The skin of his face and neck is pitted with multiple scars, which suggest smallpox; however, he explains that the scars are due to very severe acne that he had as a youngster, for which he eventually received external beam radiation therapy at the age of 14. His thyroid function tests are normal, and a fine needle aspiration (FNA) cytology of the mass is read by the pathologist as "indeterminate." Which of the following is the most appropriate next step in management?

    • A.

      No further care is needed

    • B.

      Thyroid function tests should be repeated yearly

    • C.

      Thyroid scan and sonogram are needed

    • D.

      FNA should be repeated until it can be read as benign or malignant

    • E.

      Thyroid lobectomy

    Correct Answer
    E. Thyroid lobectomy
    Explanation
    The patient is at high risk for thyroid cancer (young, male, with a single nodule and a history of radiation), and a reading of "indeterminate" in an FNA is a surgical indication. No further care (choice A) is totally wrong. It assumes that normal thyroid function means there is nothing wrong with the thyroid, when in fact thyroid cancer almost never alters thyroid function. This choice also assumes that if an FNA is not read as cancer, the patient does not have that disease. Focusing on function (choice B) as the criterion to do something is wrong for the same reasons. Thyroid scan and sonogram (choice C) were formerly valuable criteria to select surgical candidates (cold solid nodules meant a high risk of cancer), but the FNA provides a higher yield of malignancy in resected specimens, and thus has rendered the other tests obsolete for this purpose. Repeating the FNA (choice D) assumes that, given more cells, the pathologist should be able to distinguish benign from malignant. The pathologist has no trouble recognizing malignant features in papillary, medullary, or anaplastic cancers of the thyroid, but cannot do so with follicular neoplasms. Follicular adenoma and follicular carcinoma require a look at the entire specimen to tell them apart.

    Rate this question:

  • 47. 

    A 33-year-old woman is involved in a high-speed automobile collision. She arrives at the emergency department gasping for breath. Her lips are cyanotic and she has flaring nostrils. There are bruises over both sides of the chest, and tenderness suggestive of multiple rib fractures. Her blood pressure is 60/45 mm Hg, pulse is 160/min and feeble, and central venous pressure is 25 cm H2O. Her neck and forehead veins are distended. She is diaphoretic and has a hint of subcutaneous emphysema in the lower neck and upper chest. Her left hemithorax has no breath sounds and is hyperresonant to percussion. The trachea is deviated to the right, as are the heart sounds. Which of the following is the most likely diagnosis? 

    • A.

      Air embolism from tracheobronchial injuries

    • B.

      Flail chest due to multiple rib fractures

    • C.

      Massive intrapleural bleeding from torn intercostal vessels

    • D.

      Massive mediastinal bleeding from ruptured aorta

    • E.

      Tension pneumothorax caused by lung punctured by broken ribs

    Correct Answer
    E. Tension pneumothorax caused by lung punctured by broken ribs
    Explanation
    Although we typically associate tension pneumothorax with penetrating chest wounds, a blunt injury can produce lung puncture, as the jagged edges of broken ribs are driven in at the time of impact. All the classic findings of a tension pneumothorax are given in the vignette. Tracheobronchial injuries (choice A) can indeed produce subcutaneous emphysema. They can also produce air embolism if major vessels and major elements of the airway are lacerated next to one another. When that happens, though, the clinical manifestation is sudden death, typically when the patient is placed on a respirator. Flail chest (choice B) is also likely to occur with multiple rib fractures, but the clinical clue is paradoxical breathing, and the eventual problem is respiratory distress but no hemodynamic decompensation. Massive bleeding, whether from torn intercostals or ruptured aorta (choices C and D) would indeed lead to hypovolemic shock, but the central venous pressure would be zero and breathing would not be particularly affected.

    Rate this question:

  • 48. 

    A 44-year-old woman complains bitterly of severe headache that has been present for several weeks and has not responded to the usual over-the-counter headache remedies. She locates the headache to the center of her head. It is pretty much constant but is worse in the mornings. She has no other neurologic signs or symptoms. She has had "tension headaches" in the past, but she says that those were located in the back of her head and felt different from the present pain. She is currently under considerable stress because she has been unemployed since undergoing modified radical mastectomy for T3, N1, M0 breast cancer 2 years ago. She had several courses of post-operative chemotherapy, which she eventually discontinued because of the side effects. Which of the following is the most appropriate next step in diagnosis?

    • A.

      CT scan of the head

    • B.

      Psychiatric evaluation

    • C.

      Skull x-rays

    • D.

      Aortic arch arteriogram

    • E.

      Lumbar puncture

    Correct Answer
    A. CT scan of the head
    Explanation
    Persistent headache in a patient with recent history of breast cancer (particularly node-positive) is brain metastasis until proven otherwise. The only acceptable course of action is to take a look (radiologically, of course), and the cheapest and most reliable way would be a CT scan. For primary brain tumors, the MRI is favored; however, to show the presence of metastasis, an MRI is not needed. Despite the history of tension headaches and current job worries, psychiatric causes (choice B) are far down the line in the differential diagnosis. Skull x-rays (choice C) are almost a vanishing test. They may still show linear skull fractures when you are looking for them, but they would certainly not show intracranial masses. Vascular studies (choice D) were, at one time, the only reliable way to rule out intracranial tumor, but the CT scan displaced them. We still use them to define vascular lesions, which this vignette does not suggest. Lumbar puncture (choice E) would not diagnose a tumor and would subject the patient to the risk of herniation of the brainstem. Save that test for patients with meningitis.

    Rate this question:

  • 49. 

    A 59-year-old black man has a rock-hard, discrete, 1.5-cm nodule that is felt in his prostate during a routine physical examination. He is completely asymptomatic, and his prostatic specific antigen (PSA) done 3 months ago was normal for his age. His last rectal examination was performed a year earlier and was unremarkable. Which of the following will best establish the diagnosis?

    • A.

      Clinical follow-up during the ensuing year

    • B.

      Repeat determination of PSA

    • C.

      Transrectal needle biopsy of the mass

    • D.

      Transrectal sonogram of the prostate

    • E.

      Transurethral resection of the prostate

    Correct Answer
    C. Transrectal needle biopsy of the mass
    Explanation
    Cancer of the prostate will be discovered early by either the discovery of a hard nodule (as in this case), or the identification of elevated PS
    A. These are complementary examinations. One may be normal, while the other may be revealing. In this case, the recent normal PSA does not exclude the need to biopsy this mass. Clinical follow up (choice A) is inappropriate at this age, but it is not a completely stupid option: it would be the thing to do if the man had been 75. As pointed out above, regardless of PSA levels (choice B) we need to biopsy this mass. A sonogram (choice D) might be needed to identify a tumor that is not palpable, but has been discovered by the PS
    A. In this case the tumor has been felt. It can be biopsied, guided by the finger or by a sonogram if you prefer. But, the sonogram will not establish the diagnosis, it will only help do the biopsy. Let us not resect the prostate (choice E) before we have a diagnosis. Depending on the results of the complete workup, one might elect a different surgical approach or a different treatment (radiation, for instance).

    Rate this question:

  • 50. 

    A 9-month-old infant is brought in by her parents because she has an umbilical hernia. Physical examination shows an umbilical defect about 1 cm in diameter, with a small bulge when the girl cries. The hernial contents can be easily reduced. The hernia is not painful, and the girl is otherwise asymptomatic. Which of the following is the most appropriate next step in management?

    • A.

      No therapy unless the hernia persists beyond the age of 2 years

    • B.

      Repeated injections of sclerosing agents

    • C.

      Elective laparoscopic surgical repair

    • D.

      Elective open surgical repair

    • E.

      Urgent surgical repair

    Correct Answer
    A. No therapy unless the hernia persists beyond the age of 2 years
    Explanation
    Small umbilical hernias can close spontaneously up to the age of 2 years. Therefore, if they are asymptomatic and not posing an immediate risk of strangulation, they should be left alone. Obviously, every other answer offered in this question is wrong, because they all advocate aggressive therapy. However, we shall also review other ways in which they might be incorrect. Sclerosing agents (choice B) are not appropriate to manage a hernial sac that communicates with the rest of the peritoneal cavity. Occasionally, if a surgical hernia repair is done when a large distal sac has to be left in place, it might be advisable to destroy the peritoneal lining with sclerosing agents so that it does not secrete fluid. Laparoscopic hernia repair (choice C) makes sense when the size of the incision or incisions can be significantly reduced by the use of laparoscopy (for instance a bilateral inguinal hernia repair). In this case, however, one would need bigger incisions to introduce a TV camera and operating instruments than one would need to directly close a 1-cm superficial defect. Elective open surgical repair (choice D) is what this little girl will need if she still has the hernia past her second birthday. Urgent repair (choice E) would have been indicated if the hernia were tender, or if the girl had been vomiting or getting distended at the same time that the hernia became irreducible.

    Rate this question:

Quiz Review Timeline +

Our quizzes are rigorously reviewed, monitored and continuously updated by our expert board to maintain accuracy, relevance, and timeliness.

  • Current Version
  • Mar 22, 2022
    Quiz Edited by
    ProProfs Editorial Team
  • Feb 28, 2012
    Quiz Created by
    Sysplore3
Back to Top Back to top
Advertisement
×

Wait!
Here's an interesting quiz for you.

We have other quizzes matching your interest.